6
$\begingroup$

This looks extremely easy, but then again it's late at night...

Let $k$ be a commutative ring with unity. An element $a$ of a $k$-algebra $A$ is said to be transcendental over $k$ if and only if every polynomial $P\in k\left[X\right]$ (with $X$ being an indeterminate) such that $P\left(a\right)=0$ must satisfy $P=0$.

Let $n$ be a positive integer. Let $A$ be a $k$-algebra, and $t$ be an element of $A$ such that $t^n$ is transcendental over $k$. Does this yield that $t$ is transcendental over $k$ ?

There is a rather standard approach to a problem like this which works if $k$ is reduced (namely, assume that $t$ is not transcendental, take a nonzero polynomial $P$ annihilated by $t$, and consider the product $\prod\limits_\omega P\left(\omega X^{1/n}\right)$, where $\omega$ runs over a full multiset of $n$-th roots of unity adjoined to $k$; this product can be seen to lie in $k\left[x\right]$ and annihilate $t^n$; this all requires a lot more work to put on a solid footing). There are even easier proofs around when $k$ is an integral domain or a field (indeed, in this case, if $t$ is not transcendental over $k$, then $t$ is algebraic over $k$, so that, by a known theorem, $t^n$ is algebraic over $k$ as well, hence not transcendental). I am wondering if there is a counterexample in the general case or I am just blind...

$\endgroup$
4
  • 2
    $\begingroup$ Perhaps this is wrong as you say it is late: if t where not transcendental it would be algebraic so k[t] is finite dimensional over k, but k[t] contains k[t^n] so t^n is algebraic, contradiction. $\endgroup$
    – user9072
    May 29, 2013 at 2:05
  • 2
    $\begingroup$ @quid: $k$ is a commutative ring, not a field. You can't conclude that $k[t]$ is finitely generated as a $k$-module either (since we may have for example $k = \mathbb{Z}, t = \frac{1}{2}$). $\endgroup$ May 29, 2013 at 2:37
  • 5
    $\begingroup$ @darij: Let $k$ contain two elements $a, b$ such that $a^2 = ab = b^2 = 0$ and let $A = k[t]/(at^3 - b)$. By construction, $t$ is algebraic. It looks like $t^2$ might be transcendental, although I'm not sure how to prove it. $\endgroup$ May 29, 2013 at 3:03
  • $\begingroup$ @Qiaochu Yuan: but it is denoted $k$, it must be a field! :-) Thanks for the correction, somehow I thought I was missing something obvious, but that $k$ is not a field I just did not notice (while it is clearly stated). $\endgroup$
    – user9072
    May 29, 2013 at 10:25

1 Answer 1

6
$\begingroup$

$\def\F{{\mathbb F}}$ This is just a proof of Qiaochu's example.

Let $k=\F[Y,Z]/(Y^2,YZ,Z^2)$, $a=\overline{Y}$, $b=\overline{Z}$. In the ring $k[X]$ we have $I=(aX^3-b)=\{aP(X)X^3-bP(X): P(X)\in \F[X]\}$, since $a(aX^3-b)=b(aX^3-b)=0$. So each element of this ideal contains both even and odd powers of $X$. Thus in $k[X]/I$ the element $t=\overline{X}$ is algebraic, while $t^2$ is not.

$\endgroup$
3
  • $\begingroup$ I believe the term is "integral" for which one requires the equation of integrally to be monic. It might be easy, but I actually don't see why $t$ is "algebraic" over $k$. Can you tell me a polynomial $P$ in $k[x]/I$ such that $P(t) = 0$? If $\mathbb{F}$ is a field, I thought that $k[X] \cong \mathbb{F}[X,Y,Z]/(Y^2,YZ,Z^2, YX^3-Z)$. Isn't this isomorphic to $\mathbb{F}[Y,X]/ (Y^2)$? $\endgroup$
    – Youngsu
    May 29, 2013 at 8:53
  • $\begingroup$ Yes, I was wrong about the term, sorry. The polynomial is simply $at^3-b$... $\endgroup$ May 29, 2013 at 9:17
  • 1
    $\begingroup$ And yes, we may say that $k[t]={\mathbb F}[X,Y]/(Y^2)$, $t=\overline{X}$, and $k$ is a subring generated by $a=\overline{Y}$ and $b=\overline{X}^3\overline{Y}$. Then $at^3-b=0$, but $t^2$ is transcendental. $\endgroup$ May 29, 2013 at 9:22

Your Answer

By clicking “Post Your Answer”, you agree to our terms of service and acknowledge you have read our privacy policy.

Not the answer you're looking for? Browse other questions tagged or ask your own question.